Econ 202 TAMU Exam 1

Pataasin ang iyong marka sa homework at exams ngayon gamit ang Quizwiz!

Gary consumes 10,000 units of electricity when his income is $500. When his income increases to $1,000, his consumption of electricity increases to 18,000 units. What is Gary's income elasticity of demand for electricity? A) 2 B) 1.8 C) 0.5 D) 0.8

D) 0.8

Taco Bell's economists determine that the price elasticity of demand for their tacos is 2.0. So, if Taco Bell raises the price of its tacos by 6.0 percent, the quantity demanded will decrease by ________percent. A) 6.0 B) 2.0 C) 3.0 D) 12.0

D) 12.0

To calculate the price elasticity of demand we divide

the percentage change in price by the percentage change in quantity demanded. the average price by the average quantity demanded. rise by the run. the percentage change in quantity demanded by the percentage change in price. (Correct )

John is ready to pay $5 for an extra loaf of bread. Due to an ongoing discount in the store, he gets a loaf for $2. John's consumer surplus from the purchase is ________

$10 $2 $3 (Correct) $2.50

Hunter-gatherer societies allocated resources by a combination of

(1) Individual decisions. (2) Sharing customs. (3) Organized activity All of the above. (Correct) A and B above.

Advantages of trade include that

(1) It permits specialization. (2) It permits people some people to produce more than they consume. (3) It takes advantage of comparative advantage. All of the above. (Correct) A and C above.

Which of the following are assumptions needed for simple arithmetic

(1) Numbers exist. Multiplication. (2) Addition. All of the above. A and C above. (Correct)

Which of the following can be established using economic analysis

(1) People should work on teams to produce things because there is more job satisfaction. (2) Products should be made more reliably. (3) The government should not be the entity to choose the amounts of goods and services provided. None of the above can be established through economic analysis. All of the above can be established through economic analysis. (Correct )

Economics studies

(1) Scarcity. (2) The social institutions used to address scarcity. (3) How much to produce, how to produce it, and who gets it. A and C above. All of the above. (Correct)

If you have a competitive equilibrium for a two market economy consisting of tacos and enchiladas then

(1) The market equilibrium quantity of tacos is efficient. (2) The market equilibrium quantity of enchiladas is efficient. The sellers of tacos have higher costs than other potential sellers. All of the above. A and B above. (Correct )

Using basic assumptions you can determine

(1) Why markets are efficient. (2) When markets are efficient. (3) When government intervention helps or hurts economic performance. All of the above. (Correct ) A and B above.

Which of the following will lead to a change in the opportunity cost of buying a pen and a pencil?

A twofold increase in the prices of both pens and pencils An increase in the consumer's income A decrease in the consumer's income A twofold increase in the price of pens and a threefold increase in the price of pencils (Correct )

Refer to the figure above. What is the absolute value of the arc (midpoint formula) elasticity of demand when the price falls from $8 to $4? A) 2 B) 4 C) 10 D) 8

A) 2

Suppose that you are spending two hours a day studying economics, and your grade is 85 percent. You want a higher grade and decide to study for an extra hour a day. As a result, your grade risesto 90 percent. Your marginal benefit is the A) 5 point increase in your grade. B) 5 point increase in your grade minus the opportunity cost to you of spending the hour studying. C) extra hour per day you spend on studying. D) three hours per day you spend on studying.

A) 5 point increase in your grade.

Refer to figure. Which producer has an absolute advantage in producing brownies? A) Anthony B) Carlos C) Briana D) None of the above. E) All of the above.

A) Anthony

Which of the following statements is true? A) Rational economic agents maximize more than just monetary income. B) It is not necessary to consider the risks of a particular alternative while making an optimal decision. C) An individual does not require information to make optimal decisions. D) The principle of optimization is only accurate when it comes to making monetary decisions

A) Rational economic agents maximize more than just monetary income.

Soo Jin shares a one-bedroom apartment with her classmate. Her share of the rent is $700 per 5) month. She is considering moving to a studio apartment which she will not have to share withanyone. The studio apartment rents for $950 per month. Recently, you ran into Soo Jin on campusand she tells you that she has moved into the studio apartment. Soo Jin is as rational as any other person. As an economics student, you rightly conclude that A) Soo Jin figures that the additional benefit of having her own place (as opposed to sharing) is at least $250. B) Soo Jin figures that the additional benefit of having her own place (as opposed to sharing) is at least $950. C) the cost of having one's own space outweighs the benefits. D) Soo Jin did not have a choice; her roommate was a slob.

A) Soo Jin figures that the additional benefit of having her own place (as opposed to sharing) is at least $250.

Why aren't imports of roses from South America and Africa high year-round instead of just around Valentine's Day?

A) The spike in demand makes it profitable for producers with higher costs (like transportation) to enter the market. B) Flower production in the US is particularly low in February, so US-based supply is shifted leftward that month because of the climate. C) The government limits imported flowers except around Valentine's and Mothers' Day. D. A&B E. None of these

Maria has to choose between driving and taking a train to destination A. Travelling by train will cost her $400 and will take 4 hours. Driving to destination A takes 6 hours, and the required amount of gasoline costs $250. Her opportunity cost of time is $15 per hour. Refer to the scenario above. Maria should choose to: A) drive, as it will save her $120. B) drive, as it will give her a real saving of $150. C) travel by train, because it is quicker. D) travel by train, as it will save her $30 in travel time.

A) drive, as it will save her $120.

Assume that an individual spends his income on sweaters and shirts. If the price of a sweater increases: A) the opportunity cost of buying sweaters increases. B) the opportunity cost of buying shirts increases. C) the opportunity cost of buying sweaters decreases. D) There is no change in the opportunity cost of consuming either good.

A) the opportunity cost of buying sweaters increases.

A bakery can produce either cakes or cookies. If the price of cookies rises, then A) the supply curve of cake shifts leftward. B) there is a movement downward along the supply curve of cakes. C) the supply curve of cake shifts rightward. D) there is a movement upward along the supply curve of cakes.

A) the supply curve of cake shifts leftward.

After 2009, the price of tablets fell each year and manufacturers of tablets produced and sold more tablets each year. This result is because the

A. "law of demand" does not apply to customers in the "high-tech" sector of the economy. B. demand curve for tablets shifted leftward. C. supply curve of tablets shifted rightward. D. "law of supply" does not apply to companies in the "high-tech" sector of the economy.

Suppose Tinsel Town Videos lowers the price of its movie club membership by 10 percent and as a result, CineArts Videos experienced a 16 percent decline in its movie club membership. What is the value of the cross-price elasticity between the two movie club memberships?

A. -1.6 B. -0.625 C. 0.625 D. 1.6

Suppose a 4 percent increase in price results in a 2 percent increase in the quantity supplied of a good. Calculate the price elasticity of supply and characterize the product.

A. 0.5; The product is inelastic. B. 2; The product is elastic. C. 0.2; The product is inelastic. D. 50%; The product is inelastic.

Suppose that the price of a money clip increases from $0.75 to $0.90 and quantity supplied rises from 8,000 units to 10,000 units. Use the midpoint formula to calculate the price elasticity of supply.

A. 1.22 B. 1.0 C. 0.82 D. 0.07

The price elasticity of demand for Stork ice cream is -4. Suppose you're told that following a price increase, quantity demanded fell by 10 percent. What was the percentage change in price that brought about this change in quantity demanded?

A. 40 percent B. 25 percent C. 2.5 percent D. 0.4 percent

Which of the following best describes the difference between a demand curve and a demand schedule?

A. A demand curve can be derived from a demand schedule, but a demand schedule cannot be derived from a demand curve. B. A demand curve is a graphical representation of the relationship between the quantity of a good and its price, whereas a demand schedule is a tabular representation. C. A demand curve shows different quantities of a good demanded at different prices, whereas a demand schedule shows different quantities of a good demanded at different incomes. D. A demand curve shows different quantities of a good demanded at different incomes, whereas a demand schedule shows different quantities of a good demanded at different prices.

Which of the following is likely to shift the market demand curve for school textbooks to the right?

A. A fall in the price of school textbooks B. A fall in the total income of all consumers C. An increase in the enrollment rates in high schools D. An increase in school tuition fees

Which of the following is likely to cause the demand curve for cars to shift to the left?

A. A rise in the price of cars B. An increase in the cost of production leading to an increase in the price of cars C. An increase in the economy's national income D. A rise in the price of gasoline

What is the difference between an "increase in supply" and an "increase in quantity supplied"?

A. An "increase in supply" means the supply curve has shifted to the right while an "increase in quantity supplied" refers to a movement along a given supply curve in response to an increase in price. B. There is no difference between the two terms; they both refer to a shift of the supply curve. C. An "increase in supply" means the supply curve has shifted to the right while an "increase in quantity supplied" means at any given price supply has increased. D. There is no difference between the two terms; they both refer to a movement along a given supply curve.

Which of the following is the formula to calculate arc elasticity of demand?

A. Arc elasticity of demand = [(Q2 - Q1) / (Q2+ Q1)/2] / [(P2 - P1) / (P2 + P1)/2] B. Arc elasticity of demand =[(Q1 - Q2) / (Q2 + Q1)] / [(P1 - P2) / (P2 + P1)] C. Arc elasticity of demand = [(Q2 - Q1) / (Q2/2)] / [(P2 - P1) / (P2/2)] D. Arc elasticity of demand = [(Q2 + Q1) / (Q2/2)] / [(P2 + P1) / (P2/2)]

Which of the following is the correct way to describe equilibrium in a market?

A. At equilibrium, market forces no longer apply. B. At equilibrium, scarcity is eliminated. C. At equilibrium, demand equals supply. D. At equilibrium, quantity demanded equals quantity supplied.

The elasticity of demand for turkeys at Thanksgiving and roses at Valentine's day:

A. Becomes more elastic for turkeys but less elastic for roses. B. Becomes more elastic. C. Does not change. D. Becomes less elastic.

In recent years the cost of producing organic produce in the U.S. has decreased largely due technological advancement. At the same time, more and more Americans prefer organic produce over conventional produce. Which of the following best explains the effect of these events in the organic produce market?

A. Both the supply and demand curves have shifted to the right. As a result, there has been an increase in both the equilibrium price and the equilibrium quantity. B. Both the supply and demand curves have shifted to the right. As a result, there has been an increase in the equilibrium quantity and an uncertain effect on the equilibrium price. C. The supply curve has shifted to the left and the demand curve has shifted to the right. As a result, there has been an increase in the equilibrium price and an uncertain effect on the equilibrium quantity. D. The supply curve has shifted to the left and the demand curve has shifted to the right. As a result there has been an increase in the equilibrium quantity and an uncertain effect on the equilibrium price.

The cost of producing cigarettes in the U.S. has increased and at the same time, more and more Americans are choosing to not smoke cigarettes. Which of the following best explains the effect of these events in the cigarette market?

A. Both the supply and demand curves have shifted to the right. As a result, there has been an increase in the equilibrium quantity and an uncertain effect on the equilibrium price. B. Both the supply and demand curves have shifted to the left. As a result, there has been a decrease in the equilibrium quantity and an uncertain effect on the equilibrium price C. The supply curve has shifted to the right and the demand curve has shifted to the left. As a result there has been an increase in the equilibrium quantity and an uncertain effect on the equilibrium price. D. The supply curve has shifted to the right and the demand curve has shifted to the left. As a result, there has been an increase in the equilibrium price and an uncertain effect on the equilibrium quantity.

Which of the following pairs of goods is most likely to have a positive cross-price elasticity?

A. Coffee and sugar B. Printers and ink cartridges C. A privately-owned car and public transportation D. Motorcycles and typewriters

If, for a product, the quantity supplied exceeds the quantity demanded, the market price will fall until

A. Correct quantity demanded equals quantity supplied. The market price will then equal the equilibrium price. B. all consumers will be able to afford the product. C. quantity demanded equals quantity supplied. The equilibrium price will then be lower than the market price. D. the quantity demanded exceeds the quantity supplied. The market will then be in equilibrium.

Let D= demand, S = supply, P = equilibrium price, Q= equilibrium quantity. What happens in the market for walnuts if the Centers for Disease Control and Prevention announces that consuming a half cup of walnuts each week helps to lower bad levels of cholesterol?

A. D no change, S increases, P decreases, Q decreases B. S increases, D no change, P decreases, Q increases C. D and S increase, P and Q decrease D. D increases, S no change, P and Q increase

Which of the following is not an economic question:

A. Do we have enough medical care in the economy? B. Which type of music should we produce more of? C. Should we make hybrids or SUVs? D. Why do we need more Blue Bell ice cream? E. None of the above.

Which of the following statements is true of the scientific method?

A. Empirical arguments are more credible when they are based on a large data set. B. The larger the size of the data set, the greater the scope of inaccuracy in an analysis. C. Arguments based on a few anecdotes are mostly true. D. When a researcher looks at a large data set, she is more likely to jump to the wrong conclusions.

Which of the following is NOT an example of central economic control?

A. Family or clan leaders. B. Warlords. C. Dictators. D. Tribal chieftains. E. All of the above are examples of central control.

Which of the following is evidence of a shortage of walnuts?

A. Firms lower the price of walnuts. B. The quantity demanded of walnuts is greater than the quantity supplied. C. The equilibrium price of walnuts falls due to a decrease in demand. D. The price of cashews is lowered in order to make up for the walnut shortage.

Suppose the value of the price elasticity of supply is 4. What does this mean?

A. For every $1 increase in price, quantity supplied increases by 4 units. B. A 4 percent increase in the price of the good causes quantity supplied to increase by 1 percent. C. A 1 percent increase in the price of the good causes the supply curve to shift upward by 4 percent. D. A 1 percent increase in the price of the good causes quantity supplied to increase by 4 percent.

In which of the ways did the video indicate that Americans were rich compared to other societies today:

A. Housing B. Entertainment C. Food D. Medical care E. All of the above (correct)

Which of the following is NOT an economic question:

A. How many iPhones and Droids should we produce? B. Should we urbanize farmland? C. How much BlueBell should be made? D. Would people be better off if they wanted fewer goods? (correct) E. Should we teach courses online?

hunter-gatherer societies allocated resources by a combination of

A. Individual decisions. B. Sharing customs. C. Organized hunting/gathering activity D. All of the above.

With the increased usage of cell phone services, what has happened to the price elasticity of demand for land-line telephone services?

A. It has become more price elastic. B. The absolute value of the price elasticity coefficient has probably gone down. C. It has become more income elastic. D. It has become more price inelastic.

advantages of trade include that

A. It permits specialization. B. It can improve economic performance. C. It generally benefits both parties. D. All of the above.

Which of the following is a process innovation?

A. Lasik eye surgery B. smartphones C. self checkout lines D. airports

Auctions in recent years have resulted in higher prices paid for letters written by John Wilkes Booth than those written by Abraham Lincoln. What is a reason for this difference in price?

A. Many people are more fascinated by villains and anti-heroes than by heroic figures. B. Booth was a well-known actor; the demand for his letters rose as wealthy actors attempted to buy them. C. There is a surplus of letters written by Booth and a shortage of letters written by Lincoln. D. There are more letters available for collectors to buy that were written by Lincoln than there are letters that were written by Booth.

As early societies became more complex, economies started to rely

A. On custom. B. On centralized control of resources. C. On markets and trade. D. All of the above. E. None of the above.

Scarcity in an economy emerges from

A. Poor governmental decisions. B. Too many monopolies and high prices. C. Limited resources and infinite wants. D. All of the above. E. A and C above.

Why has the number of people living on farms gone down so drastically from 1900?

A. Process innovations such as herd immunization B. Investment in capital such as tractors C. Process innovations such as mechanized planting and harvesting D. All of these

Assembly lines are an example of:

A. Product innovation B. All of these C. Capital investment D. Process innovation

Which of the following could explain why the demand for table salt is inelastic?

A. Salt is a rare commodity. B. Households devote a very small portion of their income to salt purchases. C. Salt is a luxury for high-income consumers but a necessity for low-income consumers. D. Salt is a luxury good.

Which of the following are assumptions of economics?

A. Scarcity. B. Tradeoffs. C. Opportunity costs. D. All of the above. E. A and B above.

Scarcity generates which general questions in an economy:

A. Should people buy tacos or enchiladas? B. How to produce, why to produce it, and when to produce it? C. What to produce, why to produce it, and when to produce it? D. What to produce, how to produce it, and who gets it? E. B and C above.

Which of the following is NOT an economic question:

A. Should seniors get the first pick of seats in the student section at Kyle? B. Should you study for a test or party? C. Should Texas A&M allocate seats in classes using a first-come, first-served approach? D. Should you eat healthy food? E. None of the above.

Suppose at the going wage rate of $20 per hour, firms can hire as many hours of janitorial services as they desire. If any firm tries to lower the wage rate to $19, it will not be able to hire any janitor. What does this indicate about the supply curve for janitorial services?

A. Supply is relatively inelastic. B. Supply is perfectly inelastic. C. Supply is unit-elastic. D. Supply is perfectly elastic.

In October 2005, the U.S. Fish and Wildlife Service banned the importation of beluga caviar, the most prized of caviars, from the Caspian Sea. What happened in the market for caviar in the United States?

A. The demand curve shifted to the right. B. The supply curve shifted to the left. C. The supply curve shifted to the right. D. The demand curve shifted to the left.

In January, buyers of gold expect that the price of gold will rise in February. What happens in the gold market in January, holding all else constant?

A. The demand curve shifts to the left. B. The supply curve shifts to the right. C. The quantity demanded increases. D. The demand curve shifts to the right.

If the United States placed an embargo on Swedish products, what would happen in the U.S. market for Swedish furniture?

A. The demand curve would shift to the right. B. The supply curve would shift to the right. C. The demand curve would shift to the left. D. The supply curve would shift to the left.

Which of the following statements is true about the income elasticity of demand?

A. The income elasticity of demand for normal goods is always zero. B. The income elasticity of demand for inferior goods is always zero. C. The income elasticity of demand for normal goods is always positive. D. The income elasticity of demand for inferior goods is always positive.

Which of the following statements is true?

A. The more narrowly we define a market, the more elastic the demand for a product will be. B. In general, if a product has few substitutes it will have an elastic demand. C. The more time that passes the more inelastic the demand for a product becomes. D. The demand curve for a necessity is more elastic than the demand curve for a luxury.

Which of the following statements correctly differentiates between the slope of the demand curve and price elasticity of demand along a linear demand curve?

A. The price elasticity of demand for a good is the same at different points on the demand curve, whereas the slope of the demand curve varies depending on the point where it is measured. B. The price elasticity of demand is a ratio, whereas the slope of a demand curve is a product. C. The price elasticity of demand is a product, whereas the slope of a demand curve is a ratio. D. The price elasticity of demand for a good varies along the demand curve, whereas the slope of the demand curve remains the same at different points on the curve.

In February, market analysts predict that the price of titanium will rise in March. What happens in the titanium market in February, holding everything else constant?

A. The quantity of titanium demanded and the quantity of titanium supplied both increase. B. The supply curve shifts to the right. C. The demand curve shifts to the left. D. The supply curve shifts to the left.

In October, market analysts predict that the price of platinum will fall in November. What happens in the platinum market in October, holding everything else constant?

A. The supply curve shifts to the left. B. The demand curve shifts to the right. C. The quantity of platinum demanded and the quantity of platinum supplied both increase. D. The supply curve shifts to the right.

Danielle Ocean pays for monthly pool maintenance for her home swimming pool. Last week the owner of the pool service informed Danielle that he will have to raise his monthly service fee because of increases in the price of pool chemicals. How is the market for pool maintenance services affected by this?

A. There is a decrease in the supply of pool maintenance services. B. There is a decrease in the quantity of pool maintenance services supplied. C. There is a decrease in the demand for pool maintenance services. D. There is an increase in the supply of pool maintenance services.

Which of the following statements is true?

A. Total revenue will equal zero when the demand for a product is unit elastic. B. When a firm lowers its price its total revenue may either increase or decrease. C. Whenever a firm increases its quantity sold its revenue will increase. D. Whenever a firm raises its price its total revenue will increase.

A shortage occurs when the market price is lower than the equilibrium price

A. True B. False

A surplus occurs when the actual selling price is above the market equilibrium price.

A. True B. False

An increase in the quantity of a product supplied is caused by an increase in the price of the product.

A. True B. False

Because the demand for illegal drugs is inelastic and the supply is elastic, policies that reduce supply in the illegal drug market reduce revenue for drug dealers

A. True B. False

Because the supply of low-quality guns is very elastic, gun buy-back programs in a single city will reduce the number of guns by a large amount.

A. True B. False

Government policies will have their intended effect irrespective of the laws of supply and demand.

A. True B. False

In response to a surplus the market price of a good will fall; as the price falls, the quantity demanded will increase and quantity supplies will decrease until equilibrium is reached.

A. True B. False

Quantity supplied refers to the amount of a good or service that a firm is willing and able to supply at a given price.

A. True B. False

Suppose the absolute value of the price elasticity of demand for basketball game tickets on your campus is greater than 1. Increasing ticket prices will increase the total revenue from ticket sales.

A. True B. False

When Audrina raised the price of her homemade cookies, her total revenue increased. This suggests that the demand for Audrina's cookies is elastic.

A. True B. False

the additional cost to a producer of hiring an additional unit of labor is called the marginal cost.

A. True B. False

All else equal, as the price of a product falls, the quantity supplied increases.

A. True. B. False

Which of the following is an example of capital?

A. Washington, D.C. B. Kyle Field C. smartphones D. self checkout lines

Which of the following examples best describes the Law of Demand?

A. When John's income doubles, his telephone bill also doubles. B. When a new anti-tobacco commercial is released, the consumption of tobacco products decreases sharply. C. When the price of watches increases, a local manufacturer starts offering more watches for sale. D. When the price of bread doubles, John's consumption of bread halves.

Which of the following would cause an increase in the equilibrium price and an increase in the equilibrium quantity of watermelons?

A. a decrease in demand and an increase in supply B. an increase in demand and an increase in supply C. an increase in supply and an increase in demand greater than the increase in supply increase in supplyD. an increase in supply

Last year, the Pottery Palace supplied 8,000 ceramic pots at $40 each. This year, the company supplied the same quantity of ceramic pots at $55 each. Based on this evidence, The Pottery Palace has experienced

A. a decrease in supply. B. a decrease in the quantity supplied. C. an increase in the quantity supplied. D. an increase in supply.

A decrease in the demand for soft drinks due to changes in consumer tastes, accompanied by an increase in the supply of soft drinks as a result of reductions in input prices, will result in

A. a decrease in the equilibrium price of soft drinks and no change in the equilibrium quantity. B. a decrease in the equilibrium quantity of soft drinks and no change in the equilibrium price. C. a decrease in the equilibrium price of soft drinks; the equilibrium quantity may increase or decrease. D. an increase in the equilibrium quantity of soft drinks; the equilibrium price may increase or decrease.

Which of the following would cause the equilibrium price of white bread to decrease and the equilibrium quantity of white bread to increase?

A. a decrease in the price of flour B. an increase in the price of rye bread, a substitute for white bread C. an increase in the price of butter, a complement for white bread D. an increase in the price of flour

An increase in the price of off-road vehicles will result in

A. a decrease in the supply of off-road vehicles. B. a smaller quantity of off-road vehicles supplied. C. a larger quantity of off-road vehicles supplied. D. an increase in the demand for off-road vehicles.

If the demand and supply curves for a commodity both shift to the left and the shift in demand is less than the shift in supply, then in comparison to the initial equilibrium, the new equilibrium will be characterized by:

A. a lower price and a higher quantity. B. a higher price and quantity. C. the same price and quantity. D. a higher price and a lower quantity.

Assume that the supply curve for a commodity shifts to the right and the demand curve shifts to the left, both by the same degree. Then, in comparison to the initial equilibrium, the new equilibrium will be characterized by:

A. a lower price and quantity. B. a lower price and the same quantity. C. a higher price and the same quantity. D. a higher price and quantity.

Which of the following items is likely to have the highest income elasticity of demand?

A. a meal at Taco Bell B. a tank of gasoline C. a bus ride D. a vacation home in the Swiss Alps

If opportunity costs are constant, the production possibilities frontier would be graphed as

A. a negatively sloped curve bowed in toward the origin. B. a negatively sloped straight line. C. a ray from the origin. D. a positively sloped straight line.

An increase in the demand for a good is represented by:

A. a right shift to a new demand curve. B. a rightward movement along the demand curve. C. a left shift to a new demand curve. D. a leftward movement along the demand curve.

A model refers to

A. a set of facts established by observation and measurement. B. a perfect replica of reality. C. facts, measurements, or statistics that describe the world. D. a simplified description, or representation, of reality.

A decrease in the price of GPS systems will result in

A. a smaller quantity of GPS systems supplied. B. a larger quantity of GPS systems supplied. C. a decrease in the demand for GPS systems. D. an increase in the supply of GPS systems.

An increase in the price of pineapples will result in

A. a smaller quantity of pineapples supplied. B. a larger quantity of pineapples supplied. C. an increase in the supply of pineapples. D. a decrease in the demand for pineapples.

In 2004, hurricanes damaged a large portion of Florida's orange crop. As a result of this, many orange growers were not able to supply fruit to the market. Consider this new, post-hurricane situation under previous, pre-hurricane equilibrium price. We would expect to see:

A. a surplus of oranges. B. an increase in the demand for oranges. C. a shortage of oranges. D. the quantity demanded equal to the quantity supplied.

Which of the following always raises the equilibrium price?

A. an increase in demand combined with a decrease in supply B. a decrease in demand combined with an increase in supply C. an increase in both demand and supply D. a decrease in both demand and supply

Which of the following would cause a decrease in the equilibrium price and an increase in the equilibrium quantity of salmon?

A. an increase in supply B. a decrease in demand and a decrease in supply C. an increase in supply and an increase in demand greater than the increase in supply D. a decrease in demand and an increase in supply

An increase in the demand for lobster due to changes in consumer tastes, accompanied by a decrease in the supply of lobster as a result bad weather reducing the number of fishermen trapping lobster, will result in

A. an increase in the equilibrium price of lobster and no change in the equilibrium quantity. B. a decrease in the equilibrium quantity of lobster and no change in the equilibrium price. C. a decrease in the equilibrium quantity of lobster; the equilibrium price may increase or decrease. D. an increase in the equilibrium price of lobster; the equilibrium quantity may increase or decrease.

Which of the following would cause a decrease in the supply of milk?

A. an increase in the price of cookies (assuming that milk and cookies are complements) B. a decrease in the price of milk C. an increase in the number of firms that produce milk D. an increase the price of a product that producers sell instead of milk

Ranchers can raise either cattle or sheep on their land. Which of the following would cause the supply of sheep to increase?

A. an increase in the price of sheep feed B. an increase in the price of sheep C. an increase in the demand for cattle D. a decrease in the price of cattle

You have an absolute advantage whenever you

A. are better educated than someone else. B. can produce more of something than others with the same resources. C. prefer to do one particular activity. D. can produce something at a lower opportunity cost than others.

Comparative advantage means the ability to produce a good or service

A. at a lower selling price than any other producer. B. at a lower opportunity cost than any other producer. C. at a higher profit level than any other producer. D. of a higher quality than any other producer.

Economic decline (negative growth) is represented on a production possibilities frontier model by the production possibility frontier

A. becoming steeper. B. shifting outward. C. shifting inward. D. becoming flatter.

If the production possibilities frontier is ________, then opportunity costs are constant as more of one good is produced.

A. bowed out B. linear C. bowed in D. non-linear

Without an increase in the supplies of factors of production, how can a nation achieve economic growth?

A. by increasing the prices of factors of production B. by producing more high-value goods and fewer low-value goods C. by lowering the prices of factors of production D. through technological advancement which enables more output with the same quantity of resources

An article in the Wall Street Journal in early 2001 noted two developments in the market for laser eye surgery. The first development concerned side effects from the surgery, including blurred vision. The second development was that the companies renting eye-surgery machinery to doctors had reduced their charges. In the market for laser eye surgeries, these two developments

A. decreased demand and increased supply, resulting in a decrease in the equilibrium price and an uncertain effect on the equilibrium quantity of laser eye surgeries. B. decreased demand and increased supply, resulting in a decrease in both the equilibrium price and the equilibrium quantity of laser eye surgeries. C. decreased demand and increased supply resulting in an increase in both the equilibrium quantity and the equilibrium price of laser eye surgeries. D. decreased demand and decreased supply, resulting in a decrease in the equilibrium quantity and an increase in the equilibrium price of laser eye surgeries.

The equilibrium price will rise and the equilibrium quantity might increase, decrease, or stay the same when the

A. demand and the supply of a good both increase. B. demand for a good decreases and the supply of it increases. C. demand and the supply of a good both decrease. D. demand for a good increases and the supply of it decreases.

The price of turkeys goes down at Thanksgiving even though demand increases because:

A. don't actually like turkey and grocery stores know they won't buy it at higher prices. B. Supply increases by relatively more than demand because frozen turkeys are brought to market. C. Suppliers are generous and want everyone to enjoy turkeys. D. Suppliers time the production of turkeys to match up with Thanksgiving, ensuring that higher demand is met.

A service station owner in Staten Island, New York, was worried that raising the price of gasoline would cause the quantity demanded to fall by so much that he would be in a worse situation than if he did not raise the price. If raising the price of gasoline would cause the owner to receive less total revenue from the sale of gasoline, the demand for gasoline is

A. elastic. B. inelastic. C. unit elastic. D. perfectly inelastic.

If tolls on a toll road can be raised significantly before commuters will consider using a free alternative, demand for using the toll road must be

A. elastic. B. unit elastic. C. perfectly elastic. D. inelastic.

When the demand for a good decreases, its equilibrium price ________ and equilibrium quantity ________.

A. falls; decreases B. rises; decreases C. rises; increases D. falls; increases

Price elasticity of supply is used to gauge

A. how responsive suppliers are to a change in demand. B. how responsive suppliers are to changes in future prices. C. how responsive suppliers are to price changes. D. how responsive sales are to a change in input prices.

If a good has a negative income elasticity of demand, this indicates that the good is

A. inferior. B. a substitute with another good. C. normal. D. a complement with another good.

A supply schedule

A. is a table that shows the relationship between the price of a product and the quantity of the product that producers and consumers are willing to exchange. B. is a curve that shows the relationship between the price of a product and the quantity of the product supplied. C. is a table that shows the relationship between the price of a product and the quantity of the product supplied. D. is the relationship between the supply of a product and the cost of producing the product.

Every spring, motorists do more driving than during the winter months. Every spring, the price of gasoline increases and the motorists buy more gasoline. This experience suggests that the

A. laws of supply and demand are both contradicted for gasoline, though only during the spring driving season. B. "law of supply" does not always hold for necessities like gasoline. C. "law of demand" does not always hold for necessities like gasoline. D. None of the above answers are correct.

The demand for all carbonated beverages is likely to be ________ the demand for Dr. Pepper.

A. more elastic than B. less elastic than C. perfectly inelastic compared to D. perfectly elastic compared to

The production possibilities frontier shows the ________ combinations of two products that can be produced in a particular time period with available resources.

A. only B. equitable C. maximum attainable D. minimum attainable

The best alternative use of a resource is referred to as its:

A. opportunity cost. B. sunk cost. C. market price D. marginal utility.

Cross-price elasticity of demand is calculated as the

A. percentage change in quantity supplied divided by percentage change in price of a good. B. percentage change in quantity sold divided by percentage change in buyers' incomes. C. percentage change in quantity demanded of one good divided by percentage change in price of a different good. D. percentage change in quantity demanded divided by percentage change in price of a good.

If, for a given percentage decrease in price, quantity supplied decreases by a proportionately smaller percentage, then supply is

A. perfectly elastic. B. unit elastic. C. relatively inelastic. D. elastic

Total revenue equals

A. price per unit times quantity sold. B. price per unit times quantity supplied. C. change in price per unit times quantity sold. D. price per unit times change in quantity sold.

If the demand for a steak is unit elastic, then

A. quantity demanded does not respond to changes in price. B. the percentage change in quantity demanded is 100 percent greater than the percentage change in price (in absolute value). C. the percentage change in quantity demanded is equal to the percentage change in price. D. the percentage change in quantity demanded is 1 percent greater than the percentage change in price.

An increase in demand combined with no change in supply

A. raises the equilibrium price. B. results in only a movement rightward along the demand curve. C. decreases demand because the supply curve does not shift. D. lowers the equilibrium price.

If a 35 percent increase in price of golf balls led to an 42 percent decrease in quantity demanded, then the demand for golf balls is

A. relatively elastic. B. perfectly elastic. C. relatively inelastic. D. unit elastic.

If a firm raised its price and discovered that its total revenue fell, then the demand for its product is

A. relatively elastic. B. perfectly inelastic. C. relatively inelastic. D. perfectly elastic.

If firms do not increase their quantity supplied when price changes, then supply is

A. relatively inelastic. B. elastic. C. perfectly inelastic. D. perfectly elastic.

The supply curve for watches

A. shows the relationship between the quantity of watches firms are willing and able to supply and the quantity of watches consumers are willing and able to purchase. B. shows the relationship between the price of watches and the quantity of watches supplied. C. is downward sloping. D. shows the supply of watches consumers are willing and able to buy at any given price.

Suppose a medical study reveals new benefits to consuming beef and at the same time a bumper corn crop reduces the cost of feeding steers. The equilibrium price of beef will

A. stay the same. B. fall. C. rise. D. perhaps rise, fall, or stay the same, but more information is needed to determine which it does.

Suppose a decrease in the supply of bottled water results in a decrease in revenue. This indicates that

A. the demand for bottled water is elastic in the price range considered. B. the supply of bottled water is elastic in the price range considered. C. the demand for bottled water is inelastic in the price range considered. D. the supply of bottled water is inelastic in the price range considered.

If the price of lattes, a normal good you enjoy, falls

A. the income effect which causes you to increase your latte consumption outweighs the substitution effect which causes you to reduce your latte consumption, resulting in more lattes purchased. B. both the income and substitution effects lead you to buy more lattes. C. the income and substitution effects offset each other but the price effect leads you to buy more lattes. D. the substitution effect which causes you to increase your latte consumption outweighs the income effect which causes you to reduce your latte consumption, resulting in more lattes purchased.

One would speak of a change in the quantity of a good supplied, rather than a change in supply, if

A. the price of the good changes. B. prices of substitutes in production change. C. the cost of producing the good changes. D. supplier expectations about future prices change.

The producer with the least opportunity cost of picking cherries is

A. the producer who gives up the fewest apples to pick a bushel of cherries. B. the producer who can pick the most apples per bushel of cherries given up. C. the producer who can pick the most cherries. D. the producer who can pick the fewest apples. E. the producer who can pick the most apples per bushel of cherries given up AND the producer who can pick the most cherries.

If in the market for peaches the supply curve has shifted to the left,

A. the quantity of peaches supplied has increased. B. the supply of peaches has decreased. C. the quantity of peaches supplied has decreased. D. the supply of peaches has increased.

If the demand and supply curves for a commodity both shift to the left by the same amount, then in comparison to the initial equilibrium, the new equilibrium will be characterized by:

A. the same price and a higher quantity. B. a higher price quantity C. a lower price and a higher quantity. D. the same price and a lower quantity.

If, in the market for oranges, the supply has increased then

A. the supply curve for oranges has shifted to the left. B. there has been a movement downwards along the supply curve for oranges. C. the supply curve for oranges has shifted to the right. D. there has been a movement upwards along the supply curve for oranges.

In 2004, hurricanes destroyed a large portion of Florida's orange and grapefruit crops. In the market for citrus fruit

A. the supply curve shifted to the right resulting in an increase in the equilibrium price. B. the demand curve shifted to the right resulting in an increase in the equilibrium price. C. the demand curve shifted to the left resulting in a decrease in the equilibrium price. D. the supply curve shifted to the left resulting in an increase in the equilibrium price.

A change in the price of a good has two effects on the quantity consumed. What are these effects?

A. the total utility effect and marginal utility effect B. the income effect and the substitution effect C. the utility effect and the budget effect D. the consumption effect and expenditure effect

The price elasticity of supply is equal to

A. the value of the slope of the supply curve. B. the percentage change in quantity supplied divided by the percentage change in price. C. the percentage change in price divided by the percentage change in quantity supplied. D. the change in quantity supplied divided by the change in price.

marginal utility is the

A. total satisfaction received from consuming a given number of units of a product. B. average satisfaction received from consuming a product C. extra satisfaction received from consuming one more unit of a product. D. satisfaction achieved when a consumer has had enough of a product.

When demand is elastic, a fall in price causes total revenue to rise because

A. when price falls, quantity sold increases so total revenue automatically rises. B. the increase in quantity sold is large enough to offset the lower price. C. the percentage increase in quantity demanded is less than the percentage fall in price. D. the demand curve shifts.

A decrease in the equilibrium price for a product will result

A. when there is a decrease in demand and a decrease in the number of firms producing the product. B. when the quantity demanded for the product exceeds the quantity supplied C. when there is an increase in supply and a decrease in demand for the product. D. when there is a decrease in supply and a decrease in demand for the product.

A decrease in the equilibrium quantity for a product will result

A. when there is a decrease in demand and an increase in the number of firms producing the product. B. when there is a decrease in supply and a decrease in demand for the product. C. when there is an increase in supply and a decrease in demand for the product. D. when the quantity demanded for the product exceeds the quantity supplied.

If the demand for a product is perfectly inelastic, a decrease in the price of the product

A. will decrease total revenue. B. will increase total revenue. C. will not change total revenue. D. any of the above are possible.

If the percentage increase in price is 15 percent and the value of the price elasticity of demand is -3, then quantity demanded

A. will increase by 45 percent. B. will increase by 5 percent. C. will decrease by 5 percent. D. will decrease by 45 percent.

________ occurs when the direction of cause and effect is mixed up in a study.

Adverse causality Omitted variable bias Limited information bias Reverse causality (Correct)

________ is the measure of the sensitivity of one variable to a change in another.

Amplitude Buoyancy Multiplier Elasticity (Correct)

Which of the following will lead to a change in the opportunity cost of buying a pen and a pencil

An increase in the consumer's income A decrease in the consumer's income A twofold increase in the prices of both pens and pencils A twofold increase in the price of pens and a threefold increase in the price of pencils (Correct )

When people invest in a business, those investments

Are efficient if the government backs the investment. Are efficient if they prove to be successful. Are efficient if they are investing their own money.(Correct) Are often inefficient because they don't work out. B and C above.

Which of the following statements best describes a normal good? A) A normal good is a good that is readily available in the market. B) A normal good is a good whose demand increases with an increase in consumers' income. C) A normal good is a good that is rationed by the government. D) A normal good is a good whose supply increases as its price decreases.

B) A normal good is a good whose demand increases with an increase in consumers' income.

Which of the following statements is true? A) Optimizers with the lowest opportunity cost of time push up the rental price of apartments with the lowest commute time. B) As the rental prices of downtown apartments rise, only workers with the highest opportunity cost of time will be willing to rent them. C) As the rental prices of downtown apartments rise, only workers with the lowest opportunity cost of time will be willing to rent them. D) Optimizers with the highest opportunity cost of time push up the rental price of apartments with the highest commute time.

B) As the rental prices of downtown apartments rise, only workers with the highest opportunity cost of time will be willing to rent them.

Which of the following is NOT an example of an opportunity cost? A) Because David used all of his vacation time to paint his house, he was unable to visit the Caribbean last year. B) Because Mary is now being paid a higher wage, she can afford to buy a new car even though she is moving into a bigger apartment. C) By choosing to attend college, Jean was not able to continue working as an electrician; as a result, she gave up more than $85,000 in earnings while she was in college. D) By spending Thursday night studying for an economics exam, a student was unable to complete a homework assignment for calculus class.

B) Because Mary is now being paid a higher wage, she can afford to buy a new car even though she is moving into a bigger apartment.

Which of the following is TRUE? A) If consumers expect the price of a good will rise in the future, the demand curve shifts leftward. B) For an inferior good, when income increases, the demand curve shifts leftward. C) An increase in population shifts the demand curve for most goods leftward. D) The demand curve for a good shifts leftward when the price of a substitute rises.

B) For an inferior good, when income increases, the demand curve shifts leftward.

Oatmeal is a normal good and cold cereal is a substitute for oatmeal. Raisins are a complement for oatmeal. Which of the following increases the demand for oatmeal? A) an increase in the price of raisins B) an increase in the price of cold cereal C) a decrease in population D) a decrease in income

B) an increase in the price of cold cereal

If demand for Farmer John's maple syrup is inelastic, then when Farmer John raises the price of maple syrup, his total revenue will A) decrease. B) increase. C) stay the same. D) probably change, but more information is needed to determine if the total revenue increases, decreases, or stays the same.

B) increase.

In the summer 2012 the lobster catch in Maine was especially large, but instead of celebrating the fisherman were suffering from a lower total revenue. (Source: New York Times, July 28, 2012) Despitethe larger quantity of lobster caught, the total revenue of the fisherman decreased. This fact meansthat the demand for lobster is A) elastic. B) inelastic. C) perfectly elastic. D) unit elastic.

B) inelastic.

Refer to figure. Who is producing some cookies and some brownies at point D on the PPF graph on the left? A) Anthony B) Carlos C) Briana D) None of the above. E) All of the above.

C) Briana

Which of the following best describes a good with perfectly elastic demand? A) The quantity demanded of the good is completely unaffected by a price change. B) For a given price change, the percentage change in quantity demanded will be less than the percentage change in its price. C) Even the smallest increase in the price of the good will cause consumers to stop consuming it completely. D) The demand curve for the good initially slopes upward, reaches its maximum, and then slopes downward.

C) Even the smallest increase in the price of the good will cause consumers to stop consuming it completely.

A politician argues that a tornado that destroyed several houses (but caused no injuries) hassome positive impacts, because rebuilding the homes will increase employment and income for construction workers. The politician is: A) Correct, because the demand for housing construction has increased. B) Incorrect, because the supply of housing has decreased. C) Incorrect, because the opportunity cost of the resources used in rebuilding mean that other, more desirable goods can't be purchased. D) Correct, because the rebuilt homes will be more energy-efficient and have more modern appliances. E) Correct, because the construction workers will spend their additional income on other goods.

C) Incorrect, because the opportunity cost of the resources used in rebuilding mean that other, more desirable goods can't be purchased.

Which of the following is a normative statement? A) Studying more hours leads to an increase in your GPA. B) An increase in tax rates means people work fewer hours. C) States should require all motorcycle riders to wear helmets to reduce the number of riders killed. D) Taking extra vitamin C prevents catching a cold.

C) States should require all motorcycle riders to wear helmets to reduce the number of riders killed.

Which of the following correctly identifies the trade-off that a budget constraint represents? A) The optimum combination of goods that a consumer with a given income should purchase B) The maximum amount of two goods that a consumer can purchase given his income C) The amount of one good that has to be given up to purchase an additional unit of the other good D) The amount of income that must be given up to obtain an additional unit of a good

C) The amount of one good that has to be given up to purchase an additional unit of the other good

Which of the following decreases the supply of restaurant meals? A) Consumers' incomes decrease and restaurant meals are a normal good. B) The price of movies, a complement to restaurant meals, falls. C) Waiters get a pay raise. D) Consumers' incomes increase and restaurant meals are a normal good.

C) Waiters get a pay raise.

If good growing conditions increase the supply of strawberries and hot weather increases the demand for strawberries, the quantity of strawberries bought A) doesn't change and the price falls. B) doesn't change and the price rises. C) increases and the price might rise, fall or not change. D) increases and the price rises.

C) increases and the price might rise, fall or not change.

A student is studying for an exam 2 hours a day and is debating whether to study an extra hour. The student's marginal benefit: A) is greater than the student's marginal cost. B) depends on the grade the student earns on the exam. C) is the benefit the student receives from studying the extra hour. D) is the benefit the student receives from studying all 3 hours.

C) is the benefit the student receives from studying the extra hour.

One year, the government boosted regulated taxi fares in New York City by 15 percent with the expectation that the total revenue from taxi rides would also increase by 15 percent. The taxi commission that authorized this fare increase must have believed that the demand for taxi service was A) elastic, but not perfectly elastic. B) inelastic, but not perfectly inelastic. C) perfectly inelastic. D) unit elastic.

C) perfectly inelastic.

At a local ice cream parlor, when the price of half-gallons of chocolate ice cream was lowered by fifty cents per half-gallon, total revenue from the sale of chocolate ice cream decreased. This result indicates that A) there are more people who like vanilla ice cream than there are people who like chocolate ice cream. B) the demand for chocolate ice cream is elastic. C) the demand for chocolate ice cream is inelastic. D) None of the above answers is correct.

C) the demand for chocolate ice cream is inelastic.

Homer changes jobs and his new job pays him a higher income. Before he changed jobs, Homer purchased 3 pounds of tuna and 2 pounds of chicken. After he changed jobs, Homer nowpurchases 2 pounds of tuna and 3 pounds of chicken. For Homer, A) tuna is a normal good and chicken is an inferior good. B) both tuna and chicken are inferior goods. C) tuna is an inferior good and chicken is a normal good. D) both tuna and chicken are normal goods.

C) tuna is an inferior good and chicken is a normal good.

You have the choice of going on vacation to Florida for one week, staying at work for the week, or spending the week doing fix-up projects around your house. If you decide to go to Florida, the opportunity cost of the trip is A) working and doing fix-up projects. B) working, because you would be giving up income. C) working or doing fix-up projects, depending on which you would have done otherwise. D) nothing because you will enjoy the trip to Florida.

C) working or doing fix-up projects, depending on which you would have done otherwise.

When trying to assess your economic well-being you should Compare your situation to that of most of the people alive today. Compare yourself to Donald Trump. Compare yourself to your neighbors. Compare yourself to your parents. Not compare yourself to others because that is foolish.

Compare your situation to that of most of the people alive today. (Correct)

Adam Smith's Invisible Hand Theorem indicates that competitive markets are efficient because:

Competitive markets ensure efficient amounts of each good, produce that good at least cost, and allocate the good among consumers in the best way possible. (Correct) Competitive markets ensure efficient amounts of each good and produce that good at least cost Competitive markets allocate goods to consumers in the best possible way, ensure least cost production, and use prices to generate the equilibrium. Competitive markets ensure production by the least cost sellers, and are generally better than government. A and B above.

________ is the difference between the willingness to pay and the price paid for a good.

Consumer surplus (Correct) Producer surplus Revenue Seller's profit

Which of the following factors is likely to lead to an increase in the quantity demanded of pens? A) A fall in the incomes of all consumers B) A fall in the price of paper C) A rise in the incomes of all consumers D) A fall in the price of pens

D) A fall in the price of pens

Which of the following statements is true? A) Correlation can only arise when causation is not present. B) Causation arises when there is correlation between two variables, and can also arise even when there is no correlation. C) Causation can only arise when correlation is not present. D) Correlation arises when there is causation and can also arise even when there is no causation

D) Correlation arises when there is causation and can also arise even when there is no causation

Greenaqua Corp. is the only supplier of bottled drinking water in the country Lithasia. Due to the profits the firm enjoys, new corporations are interested in entering the market. If a few morecompanies producing their own line of bottled drinking water enter the market, which of thefollowing statements will be true about Greenaqua Corp.? A) The elasticity of demand of Greenaqua Corp.'s product is likely to remain the same. B) The elasticity of demand of Greenaqua Corp.'s product is likely to decrease. C) The profit Greenaqua Corp. earns on their line of bottled drinking water is likely to increase. D) The elasticity of demand of Greenaqua Corp.'s product is likely to increase.

D) The elasticity of demand of Greenaqua Corp.'s product is likely to increase.

In Module 5, Dr. Meer described the impact of local gun buyback programs in economic terms. Which of the following best describes the effects of these programs? A) The demand for guns is very inelastic, so gun buyback programs are unsuccessful at providing incentives to turn in guns. B) Because the demand for guns increases due to the buyback and the supply of guns increases due to the incentives to turn them for a fee, the price of guns goes up but the impact on the equlibrium quantity is unclear. C) The supply of guns in a city is inelastic, so increases in demand from buyback programs lead to a large number of guns removed from circulation. D) The supply of guns in a city is very elastic, so increases in demand from buyback programs lead to a large increase in the equilibrium quantity of guns in the city but small or no changes in the number of guns on the street.

D) The supply of guns in a city is very elastic, so increases in demand from buyback programs lead to a large increase in the equilibrium quantity of guns in the city but small or no changes in the number of guns on the street.

The price of cereal rises. As a result, people have cereal for breakfast on fewer days and eat eggs instead. This behavior is an example of A) an increase in the quantity demanded of eggs because of the income effect. B) an increase in the quantity supplied of eggs because of the income effect. C) a decrease in the quantity supplied of cereal because of the substitution effect. D) a decrease in the quantity demanded of cereal because of the substitution effect.

D) a decrease in the quantity demanded of cereal because of the substitution effect.

Which of the following leads to a movement along the demand curve for spinach but does NOT shift the demand curve for spinach? A) an increase in income for all spinach lovers B) an increase in the price of broccoli, a substitute for spinach C) a newly discovered health benefit from eating spinach D) a rise in the price of spinach

D) a rise in the price of spinach

Suppose a medical study reveals new benefits to consuming beef and at the same time a bumper corn crop reduces the cost of feeding steers. The equilibrium price of beef will A) fall. B) stay the same. C) rise. D) perhaps rise, fall, or stay the same, but more information is needed to determine which it does.

D) perhaps rise, fall, or stay the same, but more information is needed to determine which it does.

In March, the quantity of orange juice sold in the town of Jackson was 3000 cartons and the price $3. 37) In May, the quantity of orange juice sold in the town of Jackson was 3500 cartons and the price was$3.20. This change in the price and quantity sold could have been the result of A) the after effects of a cold winter in Florida that killed half of the orange crop. B) a reduction in the number of orange juice coupons provided by local markets. C) the after effects of a warm winter in Florida that increased the orange crop yield by 50 percent. D) the release of a medical study suggesting that consuming orange juice helps prevent cancer.

D) the release of a medical study suggesting that consuming orange juice helps prevent cancer.

Scarcity means that A. We must find ways to deal with that scarcity. B. We must make choices and give up things we really want. C. Figure out the best set of social systems to address scarcity. D. All of the above. E. A and B above.

D. All of the above. (Correct) Scarcity means that we must: (1) find ways to deal with that scarcity; (2) make choices and give up things we really want; (3) the best of social systems to address scarcity

The steps needed to evaluate the best way to organize an economy include

Determining what will be produced, how it will be produced and who gets it, and then evaluating these outcomes. (Correct) Determining whether an economy provides free choice, and an appropriate role for government. Determining what will be produced, how it will be produced and who gets it, and then limiting the role of government. Limiting central power and making the maximum use of markets. C and D above.

Which of the following is not an economic question:

Do we have enough medical care in the economy? Should we recycle more? Should we make hybrids or SUVs? Is it better for stores to close on Sundays?(Correct) All of the above are economic questions.

Refer to figure. What is the opportunity cost in terms of brownies for producing an extra cookie at point D on the left-hand graph? A) 3 B) 2 C) 1 D) 0.5 E) 0.333

E) 0.333

Refer to figure. Which producers are supplying their maximum output of brownies at point H on the graph? A) Carlos and Anthony B) Briana and Anthony C) Anthony D) All of the above. E) None of the above.

E) None of the above.

Central control of economies

Emerged with governments. Regulated resources like land and water. (Correct) Emerged when early civilizations came into existence. Was absent when society consisted of just prehistoric clans. None of the above.

Which of the following best describes a good with perfectly elastic demand?

Even the smallest increase in the price of the good will cause consumers to stop consuming it completely. (Correct) The quantity demanded of the good is completely unaffected by a price change. The demand curve for the good initially slopes upward, reaches its maximum, and then slopes downward. For a given price change, the percentage change in quantity demanded will be less than the percentage change in its price.

When voluntary exchange takes place, both parties gain from the exchange.

False True (Correct)

Which of the following is a normative economic statement?

Farmers should not be allowed to grow and sell genetically-modified crops. (Correct) The federal government is considering increasing regulations on the use of fossil fuels to promote the use of wind power. Rising corn prices have increased the price of corn-based ethanol. With rising home prices and falling mortgage interest rates, the amount of home foreclosures has decreased.

When buyers and sellers operate in a competitive market, they are

Following their own self-interest, doing whatever serves them best. (Correct) Following their own self-interest, but wanting the economy to operate efficiently. Trying to ensure that markets work. Trying to take advantage of the other person. A and C above.

Economics teaches us that people

Fundamentally do not care about others in society. Fundamentally want more goods and that is the best way to behave. Fundamentally want more goods and that is unfortunate, but real. Fundamentally want more goods.(Correct ) A and B above.

The three fundamental questions that must be answered by any economy include:

How much to produce, why goods are produced, and who gets the goods. Why to produce particular goods, how to produce them, and who gets them. How much to produce, how to produce goods, and who gets the goods. (Correct ) How to produce goods, why to produce them, and who gets them. Why to produce goods, how to produce them, and who gets them.

Which of the following is a positive economic statement?

If the price of iPhones falls, a larger quantity of iPhones will be purchased. (Correct) The U.S. government should not have bailed out U.S. auto manufacturers. The standard of living in the United States should be higher. The government should revamp the health care system.

A hunter-gatherer society

Is a notable exception to the general proposition that all societies must answer the question of what to produce. Is a notable exception to the general proposition that all societies must answer the question of how to produce it. Is a notable exception to the general proposition that all societies must answer the question of who gets it. All of the above. None of the above. (Correct)

Which of the following statements about the price elasticity of demand along a downward-sloping linear demand curve is true?

It is perfectly elastic at very high prices and perfectly inelastic at very low prices. It is inelastic at high prices and elastic at low prices. It is unit elastic throughout the demand curve. It is elastic at high prices and inelastic at low prices. (Correct)

Which of the following goods is likely to have the highest price elasticity of demand?

Ketchup (Correct) Salt Gasoline Life-saving drugs

Which of the following goods is likely to have the lowest price elasticity of demand?

Life-saving drugs (Correct) Chocolates Potato chips Decorative flowers

The key advantage of market exchange compared to one-on-one (barter) trade is that

Markets use money, which is generally better. Barter limits trades because each person must want what the other one has. Markets are more efficient. (Correct) All of the above. A and B above.

Which of the following pairs of goods are likely to be considered complements?

Motorcycles and typewriters Nokia and Samsung cell phones Pens and writing pads (Correct ) Laptops and electric heaters

The video describes the steps in simple arithmetic as proceeding from

Numbers to cardinal numbers to addition to subtraction to division to multiplication. Numbers to addition to subtraction to division to multiplication. Numbers to cardinal numbers to addition to subtraction to multiplication to division. (Correct) #s -> cardinalcardinal🐦#s -> (+) add. -> (-) sub. -> (x)mult. -> (/)div. Cardinal numbers to numbers to addition to multiplication to subtraction to division. None of the above.

Trade

Only emerged in about 1,000 B.C. Only emerged once markets became established. Has been a feature of nearly all economies. (Correct) Only came into existence when markets developed. None of the above.

Which of the following is a positive economic statement?

People should not buy SUVs. The government should mandate electric automobiles. Scarcity necessitates that people make trade-offs.(Correct ) Foreign workers should not be allowed to work for lower wages than the citizens of a country.

In which of the following ways did the video not indicate you were rich compared to King George?

Power (Correct) Indoor plumbing Travel Music Power Medical treatment

Which of the following is a positive economic statement?

Raising the tax on gasoline raises the selling price of gasoline. (Correct) The government should ban the production and sale of incandescent light bulbs. The government should revamp its immigration policies. U.S. citizens should only buy products which are produced in the United States

Which of the following is a normative economic statement?

Rising global demand for coal has led to increases in the price of coal. Pharmaceutical manufacturers should not be allowed to patent their products so prescription drugs would be more affordable. (Correct ) With rising mortgage rates and rising unemployment rates, the number of unsold homes has increased. The state of Texas is considering increasing funds for light-rail development to promote the use of public transportation

Economics is the study of

Scarcity and how societies deal with it

Which of the following can be derived from other assumptions of economics

Scarcity. (1) Tradeoffs. (1) Opportunity costs. B and C above. (Correct) A and B above.

Which of the following is not an example of the economic question of what to produce?

Should we urbanize farmland? What is the best type of cell phone?(Correct ) How much should we recycle? Should we make breakfast tacos or waffles? Should we make SUVs or all electric vehicles?

Which of the following can be derived from other assumptions about numbers

Subtraction. Multiplication. Division All of the above. (Correct) B and C above. SMD = subtraction, multiplication, division

Which of the following is a positive economic statement?

The U.S. government should increase regulations on the banking industry. If the price of beef falls, a larger quantity of it will be bought.(Correct ) The government should implement a national consumption tax. The standard of living in the United States is too low.

Adam Smith's book, the Wealth of Nations, investigated

The factors affecting economic wealth across countries. The role of wealth played in social well-being. The role of custom in the economy. The role of markets in determining economic efficiency. (Correct ) All of the above.

Which of the following would result in a higher absolute value of the price elasticity of demand for a product?

The good is a necessity. The time period under consideration is short. The expenditure on the good is small relative to one's budget. A wide variety of substitutes are available for the good (Correct )

Which of the following is a normative economic statement?

The price of gasoline is too high. (Correct) The current high price of gasoline is the result of strong worldwide demand. When the price of gasoline rises, transportation costs rise. When the price of gasoline rises, the quantity of gasoline purchased falls.

Which of the following is a normative economic statement?

The price of wheat is too low. (Correct) When the price of wheat falls, the cost of wheat-based products falls. When the price of wheat falls, the quantity of wheat purchased rises. The current low price of wheat is the result of increased worldwide supply.

In June, buyers of titanium expect that the price of titanium will fall in July. What happens in the titanium market in June, holding everything else constant?

The quantity demanded increases. The quantity demanded decreases. The demand curve shifts to the left. (Correct ) The demand curve shifts to the right.

When sellers sell goods in a market they are doing so because

They are trying to get to the efficient market solution. They want the economy to function well, so they can contribute to social welfare. They can make more money selling that good than doing something else. (Correct ) A and B above. A and C above.

A normal good is a good for which the demand increases as income decreases, holding everything else constant.

True False (Correct)

If the demand for a product is elastic, the quantity demanded changes by a smaller percentage than the percentage change in price.

True False (Correct)

If the income elasticity for canned food is 0.8, then canned food is an inferior good.

True False (Correct)

Positive analysis is concerned with "what ought to be," while normative analysis is concerned with "what is."

True False (Correct)

An inferior good is a good for which the quantity demanded decreases as the price increases, holding everything else constant

True False (Correct)

If, when price changes by 35 percent, the quantity demanded changes by 7 percent, then the absolute value of the price elasticity of demand is 5.

True False (Correct)

"An increase in the price of gasoline will increase the demand for hybrid vehicles." This statement is an example of a positive economic statement.

True (Correct) False

The price elasticity of demand for Kellogg's Raisin Bran is larger in absolute value than the price elasticity for all breakfast cereals.

True (Correct) False

A major question that faced societies 250 years ago was

What would happen if you organized more of the economy with guilds. What would happen if you organized more of the economy with markets. (Correct) What would happen if you organized more of the economy with government. Was there enough BlueBell? A, B, and C above.

Which of the following examples best describes the Law of Demand?

When the price of tea increased, the quantity demanded of tea decreased. (Correct) When Alex received a pay hike, his consumption of all goods increased. When the price of Nokia phones increased, the demand for Samsung phones increased. When the price of gasoline increased, the demand for cars fell.

Consider the following pairs of items: a. shampoo and conditioner b. iPhones and earbuds c. a laptop computer and a desktop computer d. beef and pork e. air-travel and weed killer

a and b only (Correct) c and d only e only a, b, and c only

A change in the slope of a budget constraint indicates:

a change in the consumer's tastes and preferences. a change in the consumer's income. a change in the price of either good without a change in the opportunity cost. a change in the price of either good that causes a change in the opportunity cost.(Correct )

A budget constraint is a straight line because:

a consumer faces a fixed price of both goods that do not change with changes in consumption.(Correct) the opportunity cost of buying each of the goods changes along the constraint. the tastes and preferences of the consumer change along the constraint. a consumer has a limited money income.

If, in response to an increase in the price of chocolate the quantity of chocolate demanded decreases, economists would describe this as

a decrease in demand. a decrease in quantity demanded. (Correct) a decrease in consumers' taste for chocolate. a change in consumer income.

Which of the following items is likely to have the highest income elasticity of demand?

a hamburger water breakfast cereal a luxury cruise to several European countries (Correct)

What is the difference between an "increase in demand" and an "increase in quantity demanded"?

a. An "increase in demand" is represented by a rightward shift of the demand curve while an "increase in quantity demanded" is represented by a movement along a given demand curve. (Correct) b. An "increase in demand" is represented by a movement along a given demand curve, while an "increase in quantity demanded" is represented by a rightward shift of the demand curve. c. There is no difference between the two terms; they both refer to a movement downward along a given demand curve. d. There is no difference between the two terms; they both refer to a shift of the demand curve.

Which of the following describes the substitution effect of a price change?

a. The change in quantity demanded of a good that results from a change in price, making the good more or less expensive relative to other goods, holding constant the effect of the price change on consumer purchasing power. (Correct) [...] describes the substitution effect of a price change? A change in price makes a good more or less expensive relative to other goods, resulting in a change in quantity demanded. b. The change in quantity demanded of a good that results from the change in the price of a substitute for the good. c. The change in quantity demanded of a good that results from the effect of a change in price on consumer purchasing power, holding everything else constant. d. The change in demand that results from a change in price, making the good more or less expensive relative to other goods, holding constant the effect of the price change on consumer purchasing power.

The figure above represents the market for canvas tote bags. Assume that the market price is $35. Which of the following statements is true? a. There is a surplus that will cause the price to decrease; quantity demanded will then increase and quantity supplied will decrease until the price equals $25. b. There is a surplus that will cause the price to decrease; quantity supplied will then increase and quantity demanded will decrease until the price equals $25. c. There is a surplus that will cause the price to increase; quantity demanded will then decrease and quantity supplied will increase until the price equals $25. d. There will be a surplus that will cause the price to decrease; demand will then increase and supply will decrease until the price equals $25

a. There is a surplus that will cause the price to decrease; quantity demanded will then increase and quantity supplied will decrease until the price equals $25.

See figure. Dylan has : a. a comparative advantage and an absolute advantage in making shirts. b. a comparative advantage but not an absolute advantage in making shirts. c. a comparative advantage and an absolute advantage in making pants. d. a comparative advantage but not an absolute advantage in making pants. e. none of these.

a. a comparative advantage and an absolute advantage in making shirts.

The demand curve for most goods is normally:

a. downward sloping. (Correct) b. parallel to the horizontal axis. c. upward sloping. d. parallel to the vertical axis.

The French Bakery ran a special which decreased the price of its croissants from $1.50 to $1.00. Although her money income had not changed, Toni decided to buy 2 croissants instead of her usual 1 bagel and 1 croissant. Toni's actions are explained by which of the following?

a. income and substitution effects (Correct) b. consumption effect c. price effect d. income effect only or substitution effect only but not both effects

Lily wants to maximize her benefit of consuming apples and bananas, given her fixed budget of $10 for these two fruits. The price of an apple is $1, and the price of a banana is $0.50. In order to derive Lily's demand curve for bananas, we need to ________. a. solve the buyer's problem for her multiple times and find the optimal number of bananas, when price of a bananas is at a different level each time b. draw a downward sloping line with slope of -2 c. solve the buyer's problem for her once, and find the optimal number of bananas, when price of a bananas is $0.50 d. draw a downward sloping line with slope of -1

a. solve the buyer's problem for her multiple times and find the optimal number of bananas, when price of a bananas is at a different level each time

The Law of Demand states that:

a. the quantity demanded of a commodity varies inversely with the price of the commodity. (Correct) b. the demand for a commodity always equals the supply of the commodity. c. the quantity demanded of a commodity is the same for all consumers in a perfectly competitive market. d. the demand for a commodity is mostly influenced by consumers' income.

quiz 4.5, 4.6, 4.7

all graphs

The revenue received from the sale of ________ of a product is a marginal benefit to the firm.

an additional unit (Correct) no units the total number of units only profitable units

Economic models do all of the following except

answer economic questions. portray reality in all its minute details. (Correct ) make economic ideas explicit and concrete for use by decision makers. simplify some aspect of economic life.

Economists assume that individuals

are rational and respond to incentives. (Correct) will never take actions to help others. prefer to live in a society that values fairness above all else. behave in unpredictable ways

See figure. If the price of pants is $60 then P1, the price at which the first producer will begin producing shirts is a. $15 b. $20 c. $30 d. $40 e. none of the above

b. $20

Scarcity is defined as the situation that exists when the quantity demanded for a good is greater than the quantity supplied. a. True b. False

b. False

The price elasticity of supply is equal to a. the value of the slope of the supply curve. b. the percentage change in quantity supplied divided by the percentage change in price. c. the percentage change in price divided by the percentage change in quantity supplied. d. the change in quantity supplied divided by the change in price.

b. the percentage change in quantity supplied divided by the percentage change in price. Price Elasticity of Supply PEs = (% change in quantity supplied) / (% change in price) = (ΔQ/Q) x 100 / (ΔP/P) x 100 = (ΔQ/Q) / (ΔP/P) = P(ΔQ) / Q(ΔP) = (ΔQ) / ΔP) X (P / Q) Es > 1: Elastic Es < 1: Inelastic Es = 1: Unitary Elastic

Economists assume that rational behavior is useful in explaining choices people make

because irrational people do not make economic choices. even though people may not behave rationally all the time. (Correct) even though people rarely, if ever, behave in a rational manner. because individuals act rationally all the time in all circumstances.

A good is said to have a relatively elastic demand if the value of price elasticity is:

between 0.5 and 1. between 0 and 0.5. greater than 1. (Correct ) equal to 0.

If the price of muffins, a normal good you enjoy, rises

both the income and substitution effects lead you to buy fewer muffins. (Correct) the income and substitution effects offset each other but the price effect leads you to buy fewer muffins. the substitution effect which causes you to decrease your muffin consumption outweighs the income effect which causes you to increase your muffin consumption, resulting in fewer muffins purchased. the income effect which causes you to decrease your muffin consumption outweighs the substitution effect which causes you to increase your muffin consumption, resulting in fewer muffins purchased.

We can derive the market demand curve for gold earrings

by adding vertically the quantity demanded of each gold earring consumed at each price. by adding the prices each gold earring consumer is willing to pay for each quantity. only if the tastes of all gold earring consumers are similar. by adding horizontally the individual demand curves of each gold earring consumer (Correct )

Demand for a luxury item, such as a yacht, is likely to be a. income inelastic and price elastic. b. both income inelastic and price inelastic. c. both income elastic and price elastic. d. income elastic and price inelastic.

c. both income elastic and price elastic.

If two products are complements, the ________ elasticity of demand is ________. a. income; positive b. income; negative c. cross-price; negative d. cross-price; positive

c. cross-price; negative

If two products are substitutes, the ________ elasticity of demand is ________. a. cross-price; negative b. income; positive c. cross-price; positive d. income; negative

c. cross-price; positive

Making "how much" decisions involves

calculating the total benefits of the activity and determining if you are satisfied with that amount. calculating the total costs of the activity and determining if you can afford to incur that expenditure. calculating the average benefit and the average cost of an activity to determine if it is worthwhile undertaking that activity. determining the additional benefits and the additional costs of that activity. (Correct)

Causation occurs when

change in one variable is the reason for the change in another variable. (Correct) change in one variable does not cause any change in another variable. two variables tend to move in the same direction. two variables tend to move in opposite directions.

Elvira decreased her consumption of bananas when the price of peanut butter increased. For Elvira, peanut butter and bananas are

complements in consumption.(Correct) both luxury goods. substitutes in consumption. both inferior goods.

Economists assume that the goal of consumers is to

consume as much as possible. do as little work as possible to survive. spend all their income. make themselves as well off as possible. (Correct)

The marginal product of labor is defined as a. the additional labor cost of producing one more unit of output. b. the change in total revenue that results when an additional unit of a labor is hired. c. the additional labor required to produce one more unit of output. d. the change in output that a firm produces as a result of hiring one more worker

d. the change in output that a firm produces as a result of hiring one more worker

If a consumer purchases any combination of goods and services on his ________, he will exhaust his income completely.

demand function budget constraint (Correct) indifference curve demand schedule

If a consumer purchases any combination of goods and services on his ________, he will exhaust his income completely

demand function budget constraint(Correct) indifference curve demand schedule

An omitted variable is a variable that:

does not cause other variables in a study to change when it changes. has been left out, and if included, would explain why the variables considered in a study are correlated. (Correct) is purposely left out as it does not aid an economic analysis. is removed from a study as it can lead to the problem of reverse causality.

Using Table 2, if the price of enchiladas is $3, what price of tacos would Gage require to start producing a. $2.00 b. None of the above. c. $0.50 d. $1.50 e. $1.00

e. $1.00

If at a price of $24, Octavia sells 36 home-grown orchids and at $30 she sells 24 home-grown orchids, the demand for her orchids is

elastic. (Correct) perfectly elastic. inelastic. unit elastic.

The opportunity cost of going to an outdoor music festival is

equal to the highest value of an alternative use of the time and money spent on the festival. (Correct) the cost of the festival ticket only. the value of the time spent at the festival. zero because there are no overhead costs for an outdoor festival. the enjoyment you receive from going to the festival.

In economics, the term ________ means "additional" or "extra."

equity marginal (Correct) optimal allocative

An item has utility for a consumer if it

generates enjoyment or satisfaction.(Correct) is something everyone else wants. has a high price. is scarce.

A good is said to have a relatively elastic demand if the value of price elasticity is: a. between 0 and 0.5. b. equal to 0. c. greater than 1. d. between 0.5 and 1.

greater than 1.

If a 1% change in the price of a good causes a 1% change in the quantity demanded, the good has an elasticity of demand:

greater than 1. less than 1. equal to 0. equal to 1.(Correct )

If the price elasticity of demand for insulin is equal to zero then the demand curve for insulin is

horizontal. curvilinear. vertical. (Correct) downward sloping.

Price elasticity of demand measures

how responsive quantity demanded is to a change in price. (Correct) how responsive suppliers are to price changes. how responsive sales are to a change in buyers' incomes. how responsive sales are to changes in the price of a related good.

By definition, economics is the study of

how to make money in the stock market. how to make money in a market economy. the choices people make to attain their goals, given their scarce resources. (Correct ) supply and demand

If a good has a price elasticity of demand of -3, it implies that:

if the income of the consumer increases by 1%, the quantity demanded of that good will increase by 3%. if the price of the good increases by 3%, the quantity demanded of the good will increase by 1%. if the price of the good increases by 1%, the quantity demanded of the good will decrease by 3%. (Correct ) if the income of the consumer increases by 3%, the quantity demanded of that good will increase by 1%.

The demand schedule for a commodity illustrates how the consumption of a commodity changes with changes in:

income. supply. its price. (Correct ) tastes and preferences.

If the Apple Watch and the Samsung Gear S2 are considered substitutes, then, other things equal, an increase in the price of the Apple Watch wil

increase the quantity demanded for the Gear S2. increase the quantity demanded for the Apple Watch. increase the demand for the Gear S2.(Correct ) decrease the demand for the Apple Watch.

Willingness to pay:

is equal to the price of the highest-priced goods in a consumption bundle. is equal to the price of the lowest-priced goods in a consumption bundle. is the highest price that a buyer is willing and able to pay for a unit of good. (Correct) is the lowest price that a buyer is willing and able to pay for a unit of good

Two goods are said to be substitutes when a fall in the price of one good

leads to a left shift in the demand for the other good. (Correct) leads to a right shift in the demand for the other good. doesn't affect the demand for the other good. leads to a rise in the price of the other good.

Making the buying and selling of a good illegal shifts the demand curve ________ and shifts the supply curve ________. rightward; rightward leftward; rightward rightward; leftward leftward; leftward

leftward; leftward

If demand is inelastic, the absolute value of the price elasticity of demand is

less than one. (Correct) greater than one. one. greater than the absolute value of the slope of the demand curve.

The restriction that a consumer's total expenditure on goods and services purchased cannot exceed the income available is referred to as

maximizing behavior. economizing behavior. the price constraint. the budget constraint.(Correct )

The market for smartwatches has begun to grow, due in part to the success of the Apple Watch. Following the successful launch of the Apple Watch in 2015, companies such as Samsung, Sony, and LG have all developed products to compete with the Apple Watch. The smartwatches introduced to compete with the Apple Watch would be considered

normal goods compared to the Apple Watch. complements to the Apple Watch. inferior goods compared to the Apple Watch. substitutes for the Apple Watch. (Correct)

graphs on quiz 4.10

ok

The highest valued alternative that must be given up to engage in an activity is the definition of

opportunity cost (Correct) marginal cost marginal benefit economic equity

The extra cost associated with undertaking an activity is called

opportunity cost. foregone cost. marginal cost. (Correct) net loss.

Suppose the U.S. government encouraged consumers to trade in their old automobiles for more efficient, new models by paying up to $5,000 for the old automobiles. These consumers who did trade in their old automobiles to take advantage of the government offer would be exemplifying the economic idea that

people are rational. people respond to economic incentives. (Correct) optimal decisions are made at the margin. equity is more important than efficiency.

Suppose that some investors have decided that economic and financial uncertainty have made the prospect of investing in domestic stock markets more risky than investing in foreign stock markets, and therefore choose to invest in foreign markets. By using all available information as they act to achieve their goals, these investors are exemplifying the economic idea that

people are rational. (Correct) people respond to economic incentives. optimal decisions are made at the margin. equity is more important than efficiency

Holding all other personal characteristics-such as age, gender, and income-constant, economists would expect that

people with health insurance are more likely to be overweight than people without health insurance. (Correct) people with health insurance are less likely to be overweight than people without health insurance. there is no correlation between having health insurance and being overweight. people with health insurance are equally likely to be overweight as people without health insurance.

The cross-price elasticity of demand for a good is the:

percentage change in the quantity demanded for a good due to a percentage change in the good's price. percentage change in the quantity demanded for a good due to a percentage change in tax rates. percentage change in the quantity demanded for a good due to a percentage change in the price of related goods. (Correct) percentage change in the quantity demanded for a good due to a percentage change in the consumer's income.

Facing stiff competition, Hendrix College, a small liberal arts institution in Conway, Ark., decided two years ago to bolster its academic offerings, promising students at least three hands-on experiences outside the classroom, including research, internships and service projects. Although it raised tuition and fees by 29 percent, enrollment in the freshman class rose by 37 percent.

perfectly elastic. relatively inelastic. (Correct) unit elastic. More information is needed to answer the question

When the price of tortilla chips rose by 10 percent, the quantity of tortilla chips sold fell 4 percent. This indicates that the demand for tortilla chips is

perfectly inelastic. inelastic. (Correct) unit elastic. elastic.

If quantity of milk is measured on the horizontal axis and quantity of juice is measured on the vertical axis, a decrease in the price of milk will cause the budget constraint to:

pivot rightward along the vertical axis. shift to the right. pivot rightward along the horizontal axis.(Correct ) shift to the left.

If quantity of tea is measured on the horizontal axis and quantity of coffee is measured on the vertical axis, an increase in the price of coffee will cause the budget constraint to:

pivot rightward along the vertical axis. pivot leftward along the vertical axis. (Correct) pivot rightward along the horizontal axis. pivot leftward along the horizontal axis.

As the ________ increases, ________.

price of a good; its quantity demanded decreases (Correct) quantity demanded of a good; its price increases quantity demanded of a good; its price decreases price of a good; its quantity demanded increases

The demand by all the consumers of a given good or service is the ________ for the good or service.

quantity demanded law of demand scheduled demand market demand (Correct)

Economics does not study correct or incorrect behaviors but rather it assumes that economic agents behave ________, meaning they make the best decisions given their knowledge of the costs and benefits.

rationally (Correct) equitably selfishly emotionally

A Budget Constraint

reflects the desire by consumers to increase their income. refers to the limited amount of income available to consumers to spend on goods and services.(Correct ) shows the prices that a consumer chooses to pay for products he consumes. represents the bundles of consumption that make a consumer equally happy.

Economists use the concept of ________ to measure how one economic variable, such as quantity, responds to a change in another economic variable, such as price.

relativity slope elasticity (Correct ) efficiency

Marginal utility is the

satisfaction achieved when a consumer has had enough of a product. total satisfaction received from consuming a given number of units of a product. extra satisfaction received from consuming one more unit of a product. (Correct ) average satisfaction received from consuming a product.

A decrease in the price of either good will cause a consumer's budget constraint to:

shift rightward. pivot leftward. shift leftward. pivot rightward. (Correct)

If the price of the good measured along the vertical axis increases without a change in the price of the good measured along the horizontal axis, the consumer's budget constraint:

shifts to the left. shifts to the right. pivots leftward without a change in the intercept on the horizontal axis. (Correct) pivots rightward without a change in the intercept on the horizontal axis.

If the price of a good increases, ________.

the budget constraint shifts to the left the consumer surplus decreases (Correct ) the budget constraint shifts to the right the consumer surplus increases

The income effect of an increase in the price of peaches is

the change in the quantity of peaches demanded that results from the effect of the change in price on consumer purchasing power, holding all other factors constant. (Correct) The income effect of an increase in the price of peaches is, (a) holding all other factors constant, (b) change in price effects consumer purchasing power, (c) resulting in a change in the quantity of peaches demanded. the change in the quantity of other fruit demanded that results from the impact of the price change on purchasing power, holding all other factors constant. the change in the quantity of peaches demanded that results from the price increase, making peaches more expensive than other fruit, holding constant the effect of the price change on consumer purchasing power. the change in the demand for peaches as a result of the change in the price of peaches, holding all other factors constant.

Elasticity is:

the difference of the percentage change in two variables. the product of the percentage change in two variables. the ratio of the percentage change in two variables. (Correct ) the sum of the percentage change in two variables.

The principle of opportunity cost is that

the economic cost of using a factor of production is the alternative use of that factor that is given up. the cost of production varies depending on the opportunity for technological application. the economic cost of using a factor of production is the alternative use of that factor that is given up. (Correct ) in a market economy, taking advantage of profitable opportunities involves some money cost. taking advantage of investment opportunities involves costs.

The substitution effect of an increase in the price of Raisin Bran refers to

the fact that the higher price of Raisin Bran lowers consumer's purchasing power, holding money income constant. the decrease in the demand for Raisin Bran when its price rises. the result that consumers will now switch to a substitute good such as Cheerios, and the demand curve for Raisin Bran shifts to the left. the fact that the higher price of Raisin Bran relative to its substitutes, such as Cheerios, causes consumers to buy less Raisin Bran (Correct )

When the price of audio books, a normal good, falls, causing your purchasing power to rise, you buy more of them due to

the income effect. (Correct) the substitution effect. the elasticity effect. the deadweight loss effect.

Assume that an individual spends his income on sweaters and shirts. If the price of a sweater increases

the opportunity cost of buying shirts increases. the opportunity cost of buying sweaters increases. (Correct ) the opportunity cost of buying sweaters decreases. There is no change in the opportunity cost of consuming either good.

Assume that an individual spends his income on sweaters and shirts. If the price of a sweater increases:

the opportunity cost of buying sweaters decreases. There is no change in the opportunity cost of consuming either good. the opportunity cost of buying sweaters increases. (Correct) the opportunity cost of buying shirts increases.

The slope of a budget constraint represents:

the price of the good measured along the horizontal axis. the price of the good measured along the vertical axis. the money income of the consumer. the opportunity cost of one good in terms of another (Correct )

The producer with the least opportunity cost of making a shirt is

the producer who gives up the fewest pants to make a shirt

If the demand for cell phone service is inelastic, then

the quantity demanded does not change in response to changes in price. the percentage change in quantity demanded is equal to the percentage change in price. the percentage change in quantity demanded is less than the percentage change in price (in absolute value). (Correct ) the percentage change in quantity demanded is greater than the percentage change in price (in absolute value)

The law of demand implies, holding everything else constant, that as the price of bagels increases,

the quantity of bagels demanded will increase. the demand for bagels will increase. the demand for bagels will decrease. the quantity of bagels demanded will decrease(Correct )

After a good falls in price, consumers will tend to buy more of the good that has become cheaper and less of those goods that are now relatively more expensive. This fact is called: the income effect. the substitution effect the wealth effect. the price effect.

the substitution effect

When the price of summer tank tops falls and you buy more of them because they are relatively less expensive, this is called

the substitution effect. (Correct) the income effect. the deadweight loss effect. the elasticity effect.

A correlation between two variables implies that:

there is a cause-effect relationship between the two variables. when one variable changes, the other variable always changes by exactly the same amount. it is impossible to measure one variable without measuring the other. there is a mutual relationship between both the variables. (Correct)

Economists assume that rational people do all of the following except

use all available information as they act to achieve their goals. respond to economic incentives. (Correct) undertake activities that benefit others and hurt themselves. weigh the benefits and costs of all possible alternative actions.

If demand is perfectly inelastic, the absolute value of the price elasticity of demand is

zero. (Correct) more than one. less than one. equal to the absolute value of the slope of the demand curve


Kaugnay na mga set ng pag-aaral

NCLEX Psychosocial Alterations (Mod 4)

View Set

American Gov - Test 3 Presidency and Federal Admin

View Set

Exam 261: Contact Center Analytics (5%)

View Set

Episode 11: How to Train a Brain

View Set

Child with an integumentary disorder

View Set

SS.7.C.1.4: Natural Rights and Declaration of Independence

View Set